paper 3 economy total grand test

You might also like

Download as pdf or txt
Download as pdf or txt
You are on page 1of 64

1.

Consider the following statements:

1. The responsibility of conducting socio-economic survey lie with the CSO.

2. The CSO is responsible for compilation of National Accounts

3. The NSSO conducts surveys on Consumer Expenditure, Employment-Unemployment, Social


Consumption, Manufacturing Enterprises and Service Sector Enterprises once in 10 years

4. The NSSO conducts surveys on Land and Livestock Holding and Debt and Investment once in 5
years

Which of the following statements are true?

a) 2 only

b) 1,2,3 only

c) 1,3,4 only

d) 2,1 only

Answer: a

Explanation:

1. false, it is the NSSO that is responsible for conducting socio-economic surveys.

3. false, The NSSO conducts surveys on Consumer Expenditure, Employment-Unemployment, Social


Consumption, Manufacturing Enterprises and Service Sector Enterprises once in 10 years.

4. false, The NSSO conducts surveys on Land and Livestock Holding and Debt and Investment once
every decade

2. Consider the following statements:

1.Demerit goods have positive externalities.

2.Private goods have positive externalities.

3.Public goods are both rival and excludable.

4.Giffen goods have an increased demand as prices increase.

Which of the following statements are true?

a) 1,2,3 only

b)2,3 only

c) 4 only

d) 2,4

Answer: c
Explanation: Public goods are non-rival in consumption, meaning one person's use does not diminish
its availability for others, and non-excludable, meaning everyone can use it, such as a park or
national defense.

Private goods are both rival in consumption, such as a club membership, and excludable, such as a
private house.

Demerit goods have negative externalities, such as cigarettes or alcohol, meaning they impose costs
on society beyond those who consume them.

Giffen goods may be seen as a symbol of status, and people may buy more of them as the price
increases.

3.Consider the following statements:

1. An increase in price of Veblen/Snob goods leads to a higher quantity demanded because people
perceive them to be of higher quality or status.

2. Substitute goods are goods that are used together

3. Luxury goods have an increased demand as income increases.

4. Complementary goods are goods used together

Which of the following statements are true?

A) 1,2 only

b) 1,3,4 only

c) 2,3 only

d) 3,4,1 only

Answer: b

Explanation: Veblen/Snob goods leads to a higher quantity People may believe more expensive
goods are better, leading to an increase in demand.

Substitute goods are alternatives to each other, such as tea and coffee.

Luxury goods are in higher demand as people's income increases, such as gold or designer clothes.

Complementary goods are goods used together, such as a TV and a DVD player

4.Consider the following statements:

1. Capital goods are non-durable goods used in the production process.

2. pollution is considered as merit good while education is considered as demerit good

3. DVD and tv considered as complementary good

4. Consumer goods are goods that are purchased by their ultimate consumers for final use.
5. Final goods are items that have to undergone one or more stages of production or transformation.

Which of the following statements are true?

a)1,2 only

b) 3 only

c) 1 only

d) all the above

Answer: b

Explanation: Capital goods are durable and used in the production process.

pollution is considered as demerit good while education is considered as merit good

Final goods are items that are meant for final use and will not pass through any more stages of
production or transformation.

Consumer goods are those goods that are consumed when purchased by their ultimate consumers,
such as clothes, food, and electronics.

5) Which economist is associated with the idea of laissez-faire economics?

a) Adam Smith

b) John Keynes

c) Both Adam Smith and John Keynes

d) None of the above

Answer: a) Adam Smith

Explanation: Adam Smith, a classical economist, believed that if buyers and sellers can make
decisions based on individual self-interest, it automatically ensures the welfare of the country. He
believed that the state should not intervene in the economy.

6) Which event led to the need for state intervention in the economy at a macro level?

a) Industrial Revolution

b) World War I

c) Great Depression

d) Cold War

Answer: c) Great Depression

Explanation: The Great Depression of 1929 showed the limitations of laissez-faire economics and
the need for state intervention in the economy at a macro level to address unemployment and
stabilize the economy. John Keynes, a neoclassical economist, advocated for such state intervention.
7) Which economic philosophy advocates for minimal state intervention in the economy?

a) Classical economics

b) Neoclassical economics

c) Liberal economics

d) Keynesian economics

Answer: c) Liberal economics

Explanation: Liberal economics, which includes classical and neoclassical economics, advocates for
minimal state intervention in the economy. The idea is that the market should be left to regulate
itself, and the state's role should be limited to protecting property rights, enforcing contracts, and
providing public goods.

8) Which organizations are associated with neoliberal policies?

a) IMF and World Bank

b) UN and WTO

c) OPEC and G-20

d) Red Cross and Greenpeace

Answer: a) IMF and World Bank

Explanation: Neoliberalism is an economic philosophy that emphasizes free markets, privatization,


and deregulation. The International Monetary Fund (IMF) and the World Bank are two organizations
that promote neoliberal policies, such as structural adjustment programs that require developing
countries to adopt free-market policies in exchange for loans.

9) What is the difference between Nominal GDP and Real GDP?

a) Nominal GDP is adjusted for inflation, while Real GDP is not.

b) Nominal GDP is the current year production of goods and services, while Real GDP is based on
base year prices.

c) Nominal GDP is the value of goods and services produced within a country, while Real GDP
includes exports and excludes imports.

d) Nominal GDP is used to measure economic growth, while Real GDP is used to measure inflation.

Answer: b) Nominal GDP is the current year production of goods and services, while Real GDP is
based on base year prices.

Explanation: Nominal GDP represents the value of goods and services produced in the present year
without considering inflation. On the other hand, Real GDP adjusts for inflation and is calculated
based on prices from a specific base year. Real GDP provides a more accurate measure of economic
growth over time.

10. Which economic theory believes that buyers and sellers making decisions based on individual
self-interest will automatically ensure the welfare of the country?

a) Socialism

b) Keynesian economics

c) Classical economics

d) Neoliberalism

Answer: c)

Explanation: Classical economics. Classical economics believes that individual self-interest in the
market will lead to the best allocation of resources and ultimately benefit society as a whole.

11. What are transfer payments?

a) Payments made to employees within an organization

b) Payments made to workers on transferring from one job to another

c) Payments made without any exchange of goods and services

d) None of the above

Answer: c)

Explanation: Payments made without any exchange of goods and services.

Transfer payments refer to payments made to individuals or groups without any exchange of goods
or services.

12. What is the consumption of Fixed Capital known as?

A) Depreciation

B) Capital Formation

C) Investment

D) All of the above

Answer: A

Explanation: Economically economically consumption of fixed capital, depreciation is best described


as a deduction from income to account for the loss in the capital value owning to the use of capital
goods in production
13. Which of the following is not a salient feature of Indian society?

a) Diversity of religions and cultures

b) Strong family and community values

c) Gender equality and women empowerment

d) Homogeneity of language and customs

Answer: d)

Explanation: Homogeneity of language and customs (India has a rich diversity of languages and
customs)

14. Which of the following is not a significant bottleneck in reaping India's demographic dividend?

a) Inadequate education and skills training

b) Limited job opportunities

c) Poor healthcare and nutrition

d) Rapid population growth

Answer: d)

Explanation: Rapid population growth is not a significant bottleneck in reaping India's demographic
dividend

15. Which of the following indices is used to measure the severity of hunger and undernutrition in
a country?

a) Human Development Index

b) Gender Development Index

c) Global Hunger Index

d) Multidimensional Poverty Index

Answer: c)

Explanation: According to the Global Hunger Index (GHI) 2022, India has been ranked 107th among
121 countries with a score of 29.1, placing it in the "serious" category. This represents a concerning
trend in India's hunger and malnutrition levels.

16. According to the UN World Population Prospects (WPP), 2022, which of the following is true
about India's population growth?

a) It has remained stagnant since Independence

b) It has consistently decreased since Independence


c) It has been steadily increasing since Independence

d) It has fluctuated greatly since Independence

Answer: c)

Explanation: India's population has been steadily increasing since its independence in 1947. It has
grown from 34 crores at the time of independence to 140 crores in 2022.

17. Which of the following statements are not correct?

1. According to UN World Population Prospects (WPP) 2022, India will have one of the smallest
workforces globally

2.The share of the working age population in India has grown from 50% to 65% in the last seven
decades.

3. The presence of a larger working age population and lower dependency ratio can result in
increased economic activities, which in turn can contribute to better economic growth.

Select the correct answer using the code given below:

a) 1 only

b) 1,2 only

c) 1,3 only

d) 2 only

Answer: a

Explanation: According to UN World Population Prospects (WPP) 2022, India will have one of the
largest workforces globally

In the next 25 years, one in five working-age group persons will be living in India.

18. Which of the following statements are true regarding the Total Fertility Rate (TFR) ?

1. India's TFR has declined to 4, which is below the replacement level fertility, indicating a shift in
population growth and demographic changes.

2 High illiteracy, child marriage, under-five mortality, low workforce participation, lower
contraceptive usage, and lack of women's economic say cause the uneven distribution of TFR across
Indian states.

3.The decline in TFR can have long-term impacts on India's economy, including a reduced workforce,
an aging population, and potential strains on the healthcare and social security systems.

4. Education plays a significant role in reducing fertility rates in India.

Select the correct answer using the code given below:

a) 1 only
b) 1,2 only

c) 2,3 only

d) 2 only

Answer: c

Explanation: India's TFR declined below replacement level of 2.1 children per woman in 2021 from a
high of 6 in the 1950s. While many states have achieved a TFR of 2, Bihar, UP, Jharkhand, Manipur,
and Meghalaya are still lagging.

19.Consider the following statements regarding population policy of india in 2000 (NPP-2000)
1. It is the second National Policy on Population in India, which comprehensively addresses issues
related to population growth, child survival, maternal health, women empowerment and
contraception.

2. The immediate objective of the policy is to offer service delivery in an integrated approach to
improve reproductive health and childcare.

3. The long-term objective of the policy is to achieve population stabilization in India by the year
2045.

4. The mid-term objective of the policy is to maintain a total fertility rate (TFR) of 2.1 children per
woman, which is considered as the replacement level.

Select the correct answer using the code given below:

a) None of the above

b) 1,2 only

c) 2,3 only

d) 2 only

Answer: a

Explanation: The National Population Policy, 2000 is a comprehensive document that addressed the
issue of population growth in India and aims to improve reproductive health, childcare, and achieve
population stabilization by 2045.

20. What was the aim of the second National Policy on Population launched by the government of
India in February 2000?

A. To ban birth control clinics

B. To achieve population stabilization by 2020

C. To maintain a total fertility rate of 3.0 children per woman

D. To improve reproductive health and childcare

Answer: D
Explanation: The second National Policy on Population, launched by the government of India in
February 2000, aimed to offer service delivery in an integrated approach to improve reproductive
health and childcare, maintain a total fertility rate of 2.1 children per woman, and achieve
population stabilization by 2045.

21) What was the result of using coercion and pressure during the Emergency period (1975-77) to
implement the family planning program in India?

A. The program was successfully implemented

B. The program was discredited

C. The program led to an increase in population growth

D. The program led to a decrease in population growth

Answer: B

Explanation: During the Emergency period (1975-77), coercion and pressure were used to
implement the family planning program, but this discredited the entire program.

22) which of the following statements are true regarding Radha Kamal Mukherjee committee
constituted by Indian National Congress in 1940?

1.The Radha Kamal Mukherjee Committee, appointed by the Indian National Congress in 1940, to
educate children and provide employment

2. Creating awareness about safe methods of birth control, opening birth control clinics, increasing
the age of marriage were suggested by The Radha Kamal Mukherjee Committee

Select the correct answer using the code given below:

a) None of the above

b) 1,2 only

c) 2 only

d) 1 only

Answer: c

Explanation: The Radha Kamal Mukherjee Committee, appointed by the Indian National Congress in
1940, suggested that population could be controlled by emphasizing self-control, creating awareness
about safe methods of birth control, opening birth control clinics, increasing the age of marriage,
and discouraging polygamy.

23) When did India launch its first state-sponsored family planning program?

A. 1940

B. 1951
C. 1960

D. 1974

Answer: B

Explanation: India became the first developing country to launch a state-sponsored family planning
program in 1951.

24) Which organization collaborates with NITI Aayog for the Monitoring and Analysing Food and
Agricultural Policies (MAFAP) program in India?

a) World Health Organization

b) International Monetary Fund

c) United Nations’ Food and Agriculture Organization (FAO)

d) United Nations Development Programme (UNDP)

Answer: c)

Explanation: The Monitoring and Analysing Food and Agricultural Policies (MAFAP) program in India
is a collaborative research project between NITI Aayog and the United Nations’ Food and Agriculture
Organization (FAO). It aims to monitor, analyse, and reform food and agricultural policies.

25. What is the purpose of the Village Storage Scheme conceptualized by NITI Aayog?

a) To promote natural farming in India

b) To provide storage facilities for agricultural products in rural areas

c) To improve food security in India

d) To promote the use of renewable energy in rural areas

Answer: b)

Explanation: The Village Storage Scheme conceptualized by NITI Aayog aims to provide storage
facilities for agricultural products in rural areas. It is yet to be implemented, and the Union Budget
2021 has proposed a similar scheme called Dhaanya Lakshmi Village Storage Scheme.

26. Who are the members of the NITI Aayog Governing Council?

a) Chief Ministers of all the States and Lt. Governors of Union Territories

b) Eminent experts and specialists with relevant domain knowledge

c) Members of the Council of Ministers

d) All of the above

Answer: d) All of the above.


Explanation: The NITI Aayog Governing Council comprises the Chief Ministers of all the States and Lt.
Governors of Union Territories in India, eminent experts and specialists with relevant domain
knowledge nominated by the Prime Minister, and up to four members of the Council of Ministers
nominated by the Prime Minister.

27.Consider the following statements:

1. Zero Budget Natural Farming is a farming method that does not require any monetary investment

2. Natural farming is promoted by NITI Aayog governing council and as part of the ‘Bhartiya Prakritik
Krishi Paddhati’ programme under Paramparagat Krishi Vikas Yojana (PKVY)

3. The Agriculture Vertical in NITI Aayog is a department that focuses on agriculture and fisheries
related initiatives and policies

4. The purpose of the national-level workshop on “Innovative Agriculture” organised by NITI Aayog
was to promote innovative agriculture practices and ideas

Select the incorrect answer using the code given below:

a) None of the above

b) 1,2 only

c) 3 only

d) 1 only

Answer: c

Explanation: NITI Aayog governing council and the Paramparagat Krishi Vikas Yojana (PKVY) are
promoting Zero Budget Natural Farming as "Bhartiya Prakritik Krishi Paddhati" program

The Agriculture Vertical in NITI Aayog is a department dedicated to agriculture and fisheries
initiatives and policies

The national-level workshop on "Innovative Agriculture" organized by NITI Aayog aimed to


encourage innovative agriculture practices and concepts

28) What is the aim of the Monitoring and Analysing Food and Agricultural Policies (MAFAP)
programme in India, a collaborative research project between Niti Aayog and the United Nations’
Food and Agriculture Organization (FAO)?

a) To promote organic farming among farmers

b) To monitor and analyse food and agricultural policies

c) To provide financial assistance to farmers

d) To provide training to farmers on modern farming techniques

Answer: b)
Explanation: The aim of the MAFAP programme is to monitor, analyse and reform food and
agricultural policies in India.

29) What is the Dhaanya Lakshmi Village Storage Scheme?

a) A scheme to provide financial support to rural entrepreneurs

b) A programme to promote natural farming

c) A policy to provide storage facilities to farmers for their produce

d) A research project to monitor food and agricultural policies

Answer: c)

Explanation: The Village Storage Scheme is a policy conceptualised by NITI Aayog to provide storage
facilities to farmers in rural areas to help them store their produce and prevent post-harvest losses.
The Union Budget 2021 has also proposed a similar scheme called Dhaanya Lakshmi Village Storage
Scheme, which is yet to be implemented.

30) which of the following statements are true regarding Niti Aayog?

1. The Home Minister of India is the Chairperson of NITI Aayog.

2. The Governing Council of NITI Aayog consists of the Chief Ministers of all the States and Lt.
Governors of Union Territories in India.

3. The Vice-Chairperson of NITI Aayog is appointed by the President

4. Maximum of 2 members from foremost universities, leading research organizations, and other
innovative organizations can be the part-time members of NITI Aayog

Select the correct answer using the code given below:

a) None of the above

b) 2,4 only

c) 3 only

d) 1 only

Answer: b

Explanation: The Prime Minister of India is the Chairperson of NITI Aayog

The Vice-Chairperson of NITI Aayog is appointed by the Prime Minister of India

31) According to which article of the Indian Constitution is the Union Budget referred to as the
Annual Financial Statement (AFS)?

A. Article 110

B. Article 111
C. Article 112

D. Article 113

Answer: C

Explanation: According to Article 112 of the Indian Constitution, the Union Budget of a year is
referred to as the Annual Financial Statement (AFS).

32) which of the following statements are true?

1. Gender budgeting only focuses on allocating funds for female-specific programs and schemes

2. Outcome budgeting was first introduced in India in the year 2005

3. Zero-based budgeting is a traditional approach to budgeting that has been used for many years

4. Outcome budget focuses on analysing the progress of each ministry and department in terms of
their budget outlay

Select the correct answer using the code given below:

a) None of the above

b) 2,3 only

c) 3 only

d) 1 only

Answer: b

Explanation: Zero-based budgeting is a relatively new approach to budgeting that gained popularity
in the 1970s

Gender budgeting aims to allocate funds for development, welfare, and empowerment schemes and
programs that promote gender equity, not just female-specific programs

Outcome budget measures the development outcomes of all government programs and analyzes
what each ministry has done with its budget outlay

33) Which of the following is an example of revenue receipt?

a) Investment in infrastructure

b) Loan from the World Bank

c) Dividend income from public sector undertakings

d) Repayment of a loan to public banks

Answer: c) Dividend income from public sector undertakings.


Explanation: Revenue receipts are receipts which do not have a direct impact on the assets and
liabilities of the government. Examples include tax revenue, dividend income, profits, and interest
receipts.

34) Which of the following is not a type of capital receipt?

a) Selling assets

b) Borrowings

c) Repayment of loans

d) Tax revenue

Answer: d)

Explanation: Capital receipts indicate the receipts which lead to a decrease in assets or an increase
in liabilities of the government, such as the money earned by selling assets (or disinvestment) such
as shares of public enterprises, and the money received in the form of borrowings or repayment of
loans by states. Tax revenue is a type of revenue receipt.

35) What are some of the issues with agricultural subsidies in India?

a. Indiscriminate subsidy use can create inefficiencies

b. Less focus on micronutrients can cause an imbalance of soil nutrients and deterioration of soil
quality

c. Most indirect subsidies on fertilizers, power, and irrigation water contribute to the degradation of
natural resources

d. All of the above

Answer: d

Explanation: Some of the issues with agricultural subsidies in India include their contribution to the
degradation of natural resources, distortion of cropping patterns, pricing issues, less focus on
micronutrients, compromising investment in capital infrastructure, fiscal burden to states, and
inefficiencies due to fraud, diversion, and waste.

36) Consider the following statements regarding the Pradhan Mantri Fasal Bima Yojana (PMFBY):

1. The scheme also aims to encourage farmers to adopt innovative and modern agricultural practices
to improve productivity and reduce crop losses

2. Pradhan Mantri Fasal Bima Yojana (PMFBY) is a crop insurance scheme launched by the
Government of India in 2016

3. The remaining premium is shared between the central and state governments in a 70:30 ratio

4. The scheme covers risks leading to crop loss such as yield losses, prevented sowing, post-harvest
losses, and localized calamities
Select the correct answer using the code given below:

a) None of the above

b) 2,3 only

c) 3 only

d) 1,2,4 only

Answer: d

Explanation: The Pradhan Mantri Fasal Bima Yojana (PMFBY) aims to support sustainable agriculture
by providing financial support to farmers in case of crop loss or damage due to unforeseen events.
The scheme covers yield losses, prevented sowing, post-harvest losses, and localized calamities on
individual farm or notified area basis. The premium rates for the scheme are fixed at 2% for all Kharif
crops, 1.5% for all Rabi crops, and 5% for annual commercial and horticultural crops. Shared
between state and center 50:50

37) Which agency works under the aegis of Ministry of Jal Shakti to receive Central Share under
LTIF?

a) National Water Development Agency (NWDA)

b) National Bank for Agriculture and Rural Development (NABARD)

c) State Governments

d) Reserve Bank of India (RBI)

Answer: a) National Water Development Agency (NWDA)

Explanation: NABARD offers loans for both Central Share and State Share under the Long Term
Irrigation Fund (LTIF) for a period of 15 years. The Central Share portion of the loan is given to the
National Water Development Agency (NWDA), which operates under the Ministry of Jal Shakti. On
the other hand, the State Governments avail the loan for the State Share portion

38) What is Long Term Irrigation Fund (LTIF)?

a) A fund to provide financial assistance to farmers for crop insurance

b) A fund to provide loans for small scale irrigation projects

c) A fund to fast-track the completion of medium and major irrigation projects

d) A fund to provide subsidies for drip irrigation systems

Answer: c) A fund to fast-track the completion of medium and major irrigation projects

Explanation: Long Term Irrigation Fund (LTIF) was operationalized in the National Bank for
Agriculture and Rural Development (NABARD) during 2016-17 for fast tracking the completion of the
identified Medium and Major Irrigation projects, spread across different states.
39) Which of the following crops are covered under minimum support price (MSP)?
1. Paddy
2. Wheat
3. Cotton
4. Jute
5. Coffee
Select the correct answer using the code given below:

a) 1,2,3,4,5 only

b)1,2,3,4 only

c) 1,3,5 only

d)1,2,3,4only

Answer: d

Explanation: Twenty two crops covered under MSP are Paddy, Jowar, Bajra, Maize, Ragi, Arhar,
Moong, Urad, Groundnut-in-shell, Soyabean, Sunflower, Seasamum, Nigerseed, Cotton, Wheat,
Barley, Gram, Masur (lentil), Rapeseed/Mustardseed, Safflower, Jute and Copra.

40) Which organizations are responsible for purchasing agricultural products?


1. Food Corporation of India (FCI).
2. Cotton Corporation of India (CCI).
3. National Consumer Cooperative Federation of India Ltd. (NCCF).

4. food and agricultural association


Select the correct answer using the code given below:

a) 1,2,3,4 only

b)1,2,3 only

c) 1,3 only

d)1,2,4only

Answer :b

Explanation: The Government carries out the procurement of agricultural commodities using
different public and cooperative agencies, including Food Corporation of India (FCI), Cotton
Corporation of India (CCI), Jute Corporation of India (JCI), Central Warehousing Corporation (CWC),
National Agricultural Cooperative Marketing Federation of India Ltd. (NAFED), National Consumer
Cooperative Federation of India Ltd. (NCCF), and Small Farmers Agro Consortium (SFAC)

41) How is the Minimum Selling Price (MSP) of white/refined sugar determined in India?

a) Based on the components of Minimum Support Price (MSP) of sugarcane

b) Based on the recommendations of the Ministry of Agriculture and Farmers Welfare


c) Based on the conversion cost of the most efficient mills

d) Based on market-driven prices of sugar

Answer: c) Based on the conversion cost of the most efficient mills

Explanation: The Government sets a minimum selling price (MSP) for white/refined sugar sold by
sugar mills at the factory gate for domestic consumption. The MSP is determined based on the
components of Fair & Remunerative Price (FRP) of sugarcane and minimum conversion cost of the
most efficient mills. This ensures a fair price for both the sugar mills and sugarcane farmers.

42) What is the basis for determining the Fair and Remunerative Price (FRP) of sugarcane in India?

a) Recommendations of the Ministry of Agriculture and Farmers Welfare

b) Consultation with the sugar industry associations only

c) Recommendations of the Commission for Agricultural Costs and Prices (CACP) in consultation with
the State Governments and feedback from sugar industry associations

d) Market-driven prices based on demand and supply

Answer: c)

Explanation: Recommendations of the Commission for Agricultural Costs and Prices (CACP) in
consultation with the State Governments and feedback from sugar industry associations

43) What is the main objective of Minimum Support Price (MSP) for agricultural produce?

A. To increase competition among farmers

B. To guarantee prices and market to farmers

C. To decrease the supply of agricultural produce

D. To encourage farmers to shift to non-agricultural sectors

Answer: B.

Explanation: The main objective of MSP is to give guaranteed prices and assured market to the
farmers and save them from the price fluctuations.

44) What is the difference between Fair and Remunerative Price (FRP) and Minimum Selling Price
(MSP) in the context of sugarcane and sugar industries?

A. FRP is for sugarcane and MSP is for sugar

B. MSP is for sugarcane and FRP is for sugar

C. Both are the same and used interchangeably

D. None of the above


Answer: A

Explanation: The government announces Fair and Remunerative Price (FRP) for sugarcane, while
Minimum Selling Price (MSP) is fixed for white/refined sugar for sale by sugar mills at the factory
gate for domestic consumption, taking into account the components of FRP of sugarcane and
minimum conversion cost of the most efficient mills.

45) Which of the following statements about farm subsidies is/are true?

A. Developed countries provide higher farm subsidies than developing countries like India.

B. The World Bank has set limits on the number of subsidies provided by developed and developing
nations.

Choose the correct answer:

A. Only statement 1 is true

B. Only statement 2 is true

C. Both statements 1 and 2 are true

D. Neither statement 1 nor 2 is true

Answer: A

Explanation: Farm subsidies are a significant part of government budgets, with developed countries
allocating about 40% of their total budgetary outlay to them. However, the World Trade
Organization has imposed limits on direct and indirect subsidies to prevent market distortion.

46) Which of the following statements is/are correct regarding the Production Linked Incentive
(PLI) Scheme for Large Scale Electronics Manufacturing?

1.The scheme proposes a financial incentive to boost domestic manufacturing in the electronics
value chain.

2.The scheme is applicable to all segments of the electronics industry.

3. The scheme covers all electronics items manufacturing in India except mobile phones.

A. 1 only

B. 1,2 only

C. 2,3 only

D. Neither statement 1 nor 2 is correct

Answer: A

Explanation: The PLI scheme for Large Scale Electronics Manufacturing is aimed at promoting
domestic manufacturing and attracting investments in the electronics industry. However, it is
applicable only to specific target segments including mobile phones and specified electronic
components
47) What does a PMI (The purchasing managers’ index) figure above 50 indicate?

a) Expansion in business activity

b) Contraction in business activity

c) No change in business activity

d) None of the above

Answer: a)

Explanation: A PMI figure above 50 indicates that there has been an expansion in business activity
compared to the previous month. The higher the PMI figure above 50, the greater the level of
expansion. Conversely, a PMI figure below 50 indicates a contraction in business activity.

48) Who conducts the Annual Survey of Industries (ASI) in India?

a) Reserve Bank of India (RBI)

b) National Sample Survey Office (NSSO)

c) National Statistical Office (NSO)

d) Ministry of Finance

Answer: c)

Explanation: The Annual Survey of Industries (ASI) is conducted by the National Statistical Office
(NSO), Ministry of Statistics & Programme Implementation.

49) What is the purpose of the Annual Survey of Industries (ASI)?

a) To provide statistical information on agricultural production in India

b) To provide information on the growth of the services sector in India

c) To provide information on the growth, composition and structure of organized manufacturing


sector in India

d) To provide information on the financial sector in India

Answer: c)

Explanation: The Annual Survey of Industries (ASI) is the principal source of industrial statistics in
India. It provides statistical information to assess and evaluate, objectively and realistically, the
changes in the growth, composition and structure of organized manufacturing sector comprising
activities related to manufacturing processes, repair services, gas and water supply and cold storage.

50) What is the National GIS-enabled land bank system launched by the Commerce and Industry
Minister?
a. A system to track the movement of industrial goods

b. A system to monitor environmental impact of industries

c. A system to provide real-time information about availability of industrial land and resources

d. A system to regulate the prices of industrial land

Answer: c

Explanation: The National GIS-enabled land bank system launched by the Commerce and Industry
Minister is a prototype system that will provide real-time information to investors about the
availability of industrial land and resources. The system is being developed by integration of
Industrial Information System (IIS) with State GIS Systems.

------------------

51. Consider the following statements:


1. The Dalit Bandhu scheme is aimed at promoting entrepreneurship among Dalits

2. The Telangana Dalit Bandhu initiative provides loans to scheduled caste families to establish
income generating schemes

3. Under the scheme, Dalit families receive a direct benefit transfer of Rs 10 lakh each

Which of the following statements are true:

a) 1,2 only

b) 1,3 only

c) 1 only

d) 1,2,3 only

Answer: b

Explanation: The Telangana Dalit Bandhu initiative provides a one-time capital assistance of Rs.10
lakhs per SC family as a 100% grant/subsidy to establish a suitable income generating scheme as per
their choice, without bank loan linkage

Scheme is aimed at promoting entrepreneurship among Dalits.

52. Consider the following statements regarding Rythu Bima Scheme:

1. Individuals who are between the ages of 18 and 59 are qualified to participate in the program

2.The entire premium is paid by the government to the Life Insurance Corporation of India

3. The primary aim of the Farmers Group Life Insurance Scheme ( Rythu Bima) is to offer economic
support and social protection to farmers in the event of Crop loss due to any cause
4. Agriculture in Telangana State faces challenges due to frequent droughts, poor investment
capacity of farmers.

Which of the following statements are not true:

a) 1,2,3 only

b) 2,3 only

c) 1,2,4 only

d) 3 only

Answer: d

Explanation: The primary aim of the Farmers Group Life Insurance Scheme (Rythu Bima) is to offer
economic support and social protection to farmers in the event of the loss of the farmer's life,
regardless of the cause.

53. Which of the following statements correctly describe the objective of “Rythu Bandhu Scheme”?

1. Under the Rythu Bandhu Scheme, farmers receive a grant of Rs. 4,000/- per acre per season to
purchase inputs, and other investments for field operations related to the crop season.

2. Government of Telangana allocated a budget of Rs. 10,000 Crores for the Agriculture Investment
Support Scheme for the financial year 2018-19.

3. The Agriculture Investment Support Scheme is proposed to be implemented from the year 2022-
23

Which of the following statements are not true:

A) 1,2 only

B) 2,3 only

C) 1,2,3 only

D) 3 only

Answer: C

Explanation: The Agriculture Investment Support Scheme was proposed to be implemented from the
year 2018-19

The Investment Support Scheme for the financial year was Rs.12,000 Crores

farmers receive a grant of Rs. 5,000/- per acre per season to purchase inputs, and other investments.

54. Which of the following is a limitation of the Gross Domestic Product?

a. It fails to indicate the sustainability of a nation’s growth

b. It doesn’t consider the impact of economic activities on human health and the environment
c. It does not include non-market transactions

d. All of the above

Answer: d

Explanation:

GDP is quantitative in nature. It ignores the qualitative aspect of the economy. Other limitations of
GDP are

Social indicators like health education etc not taken into consideration.

Informal sector not of GDP.

Only monetary transactions calculated.

Impact on environment neglected.

55. Which of the following statements are true?

1. The Kalyana Lakshmi Shaadi Mubarak scheme provided financial assistance to minorities only

2. BCs received the highest percentage of financial assistance under the scheme

3. SCs received the lowest percentage of financial assistance under the scheme

4. The scheme was implemented only from 2019-20 to 2020-21

A) 1,2 only

B) 2,3 only

C) 2 only

D) 3 only

Answer: C

Explanation :Kalyana Lakshmi Shaadi Mubarak scheme provided financial assistance to SC, ST, BC’s,
EBC’s

The project was started in 2014 since then govt of Telangana providing grants

Not SCs but Minorities received the lowest percentage of financial assistance under the scheme

56) Consider the following statements:

1. The Mahatma Gandhi National Rural Employment Guarantee scheme guarantees 50 days of
unscaled wage employment in a financial year to a rural household

2. The execution of works under MGNREGA is carried out by the State Government

3. The Government of Telangana launched the Palle Pragati program in 2019 to create additional
productive assets in urban areas
4. The Palle Pragati program is providing employment and creating permanent and quality assets in
rural areas.

Which of the following statements are not true:

A) 1,2 only

B) 2,3 only

C) 1,2,3 only

D) 4 only

Answer: C

Explanation :MGNREGA scheme guarantees 100 days of unscaled wage employment in a financial
year to a rural household

The execution of works is carried out by the Panchayati Raj Institutions

The program was launched to improve the quality of life in rural areas

57) What is the primary objective of the Palle Pragati program?

a) To Improve the quality of Education in rural areas

b) To provide employment and create additional productive assets

c) To reduce the governance of Gram Panchayats

d) To channelize MGNREGA for urban areas

Answer: b)

Explanation: To provide employment and create additional productive assets Palle Pragati program
was launched in the year 2019 on 6th September

58) What is the status of the Haritha Haram program in 2021?

a) In the 4th phase

b) In the 5th phase

c) In the 6th phase

d) The program has been discontinued

Answer: c)

Explanation :Haritha Haram program was started in July 2015 the 6th phase of TKHH started in 2020
from Medak

59) Which fund was established by the Telangana government to receive contributions for the
Haritha Haram program?
a) Telangana Green Fund

b) Telangana Clean Fund

c) Telangana Forest Fund

d) Telangana Tree Fund

Answer: a)

Explanation: The government has established Telangana Haritha Nidhi or Telangana green fund That
receives contribution from public representatives government employees and civil society

60) What is the primary approach of the Haritha Haram program to achieve the 33% forest cover in
Telangana?

a) Rejuvenation of degraded forests

b) Prevention of smuggling of forest produce and forest encroachment

c) Soil and moisture conservation measures

d) All the above

Answer: d)

Explanation :A three-pronged approach, including rejuvenation of degraded forests, prevention of


smuggling of forest produce and forest encroachment, and soil and moisture conservation measures

61) Consider the following statements:

1. Industrial needs get a 50% share of the total water provided by Mission BhagIratha

2. Mission Bhagiratha aims to provide water supply to all rural habitations through functional tap
connections

3. The project provides treated drinking water at the rate of 100 liters per capita per day in rural
areas and 150 liters per capita per day in municipal corporations

Which of the following statements are not true:

A) 1,2 only

B) 2,3 only

C) 1,2,3 only

D) 3 only

Answer: A

Explanation :Industrial needs get a 10% share of the total water provided by Mission Bhagiratha,
while the remaining water is earmarked for rural habitations.
Mission Bhagiratha aims to provide water supply to all rural habitations through functional tap
connections, besides providing bulk water supply to the urban local bodies.

The project provides treated drinking water at the rate of 100 liters per capita per day in rural areas,
135 liters per capita per day in municipality Nagar.

62) Consider the following statements Regarding Self Help Groups in Telangana:

1. The Telangana government promotes agriculture-related activities through SHGs to enhance the
income-earning capacities of women.

2. The Society for Elimination of Rural Poverty is a privately funded NGO in Telangana.

3. Telangana's SHGs government has emerged as the world's largest and most successful network of
community-based organizations

Which of the following statements are not true:

A) 2 only

B) 2,3 only

C) 1,2,3 only

D) 3 only

Answer: A

: Explanation :It is a non-profit organization promoted by the government to alleviate poverty in


rural areas through self-help groups (SHGs).

The Telangana government is promoting various agriculture-related activities through SHGs to


enhance the income-earning capacities of women. These activities include paddy procurement,
farmer producer group formation, and livestock rearing.

63) Consider the following statements referring to Ryotwari systems:

1. The farmers whose names were not registered in village registers were called as Shikmidars

2. According to the Hyderabad land revenue act 1908, state had complete and ultimate ownership
right on the land.

3. Asami Shikmis are tenant farmers who lease out the land to cultivate.

Which of the following statements are true:

A) 2 only

B) 2,3 only

C) 1,2,3 only

D) 3 only

Answer: C
Explanation :Shikmidars were farmers who worked on land but were not listed as landowners in
village records. The Hyderabad land revenue act of 1908 gave the state complete ownership rights
over the land, while Asami Shikmis were tenants who leased land from landowners to cultivate.

64) What is the significance of Tandur red gram receiving a Geographical Indication (GI) tag?

a) It will increase the demand for red gram in Telangana

b) It will help farmers boost their incomes

c) It will provide an opportunity for farmers to diversify their crops

d) All of the above

Answer: d)

Explanation: Tandur red gram receiving a Geographical Indication (GI) tag will help farmers boost
their incomes, provide an opportunity for crop diversification, and increase demand for red gram in
Telangana.

65.What is the state's rank in terms of Oil Extraction Rate (OER) in India?

a) First

b) Second

c) Third

d) Fourth

Answer: a) First

Explanation: Telangana is ranked first in Oil Extraction Rate (OER) in India.

66. Which crop has seen a significant increase in production due to the government's investment in
irrigation projects?

a) Oil palm

b) Paddy

c) Red gram

d) None of the above

Answer: b)

Explanation: The government's investment in irrigation projects has led to a significant increase in
paddy production in Telangana.

67. Which of the following statements are correct regarding oil palm?
1. The Telangana government is not providing any assistance for taking up oil palm cultivation

2. The Telangana government is planning to take up 20 lakh acres under oil palm cultivation in a
mission mode as part of crop diversification

3. Telangana stands sixth in the country in terms of the area under oil-palm cultivation.

Which of the following statements are true:

A) 2 only

B) 2,3 only

C) 1,2,3 only

D) 3 only

Answer: B

The Telangana government aims to diversify crops and plans to cultivate 20 lakh acres of oil palm in
a mission mode, with 50% assistance on mechanisation tools. The state stands sixth in the country in
terms of the area under oil-palm cultivation.

68) Consider the following statements:

1. Tandur red gram has received a GI tag, making it the 16th product from Telangana to receive this
recognition.

2. The Telangana Civil Supplies Corporation Limited has purchased huge quantities of paddy from
farmers, ensuring the Minimum Support Price.

3. The Telangana government is supporting a transition to demand-driven and commercial


agriculture, which will provide a sustainable increase in incomes to farmers.

Which of the following statements are true:

A) 2 only

B) 2,3 only

C) 1,2,3 only

D) 3 only

Answer: C

Explanation :Tandur red gram, a crop from Telangana, has been granted a GI tag, becoming the 16th
product from the state to receive this recognition. Additionally, the Telangana government is
promoting a shift towards commercial and demand-driven agriculture, leading to sustainable income
growth for farmers.

69) Consider the following statements:

1. Assured irrigation is not an important input for crop production.


2. The Telangana government has invested a considerable amount in irrigation projects from 2014-
15 to 2022-23, leading to a significant increase in the Gross Irrigated Area.

3. The paddy production in Telangana has decreased between 2015-16 and 2021-22.

Which of the following statements are true:

A) 2 only

B) 2,3 only

C) 1,2,3 only

D) 3 only

Answer: A

Explanation :Assured irrigation is the main input for crops as it protects farmers in poor

Paddy production in Telangana has increased by 342% between 2015-16 and 2021-22

70) Which sector has been the main driver of growth in Telangana's agriculture sector from 2014-15
to 2022-23?

a. Crops

b. Forestry and logging

c. Fishing and aquaculture

d. Livestock

Answer: d.

Explanation: Livestock accounts for 47.69% of the overall sector Gross Value Added (GSVA) at
current prices, making it the main driver of growth in Telangana's agriculture sector.

71) What percentage of Telangana's geographical area is under forest cover?

A) 7.46%

B) 24.70%

C) 5.26%

D) 5.42%

Answer: B)

Explanation: As per the given information, around 24.70% of Telangana's geographical area is under
forest cover.

72) What is the average land holding size in Telangana as per the Agriculture Census, 2015-16?
A) 1.08 hectares

B) 1.00 hectare

C) 4.94 acres

D) 59.48 lakhs

Answer: B)

Explanation: As per the given information, the average land holding size in Telangana as per the
Agriculture Census, 2015-16, is 1.00 hectare, which is below the all India average of 1.08 hectares.

73) Consider the following statements:

1. Horticulture crop sector contributes 31% to Agriculture GVA in terms of value of the produce in
the State

2. The area under horticulture crops has decreased over the years

3. The horticulture sector has contributed significantly to the overall growth of the Agriculture &
Allied sector in the State

Which of the following statements are true:

A) 2 only

B) 2,3 only

C) 1,2,3 only

D) 3 only

Answer: A

Explanation :The horticulture sector has been a driving force in the overall growth of the Agriculture
& Allied sector in the State

The given information states that the area under horticulture crops has been increased by 240%
from 3.80 lakh acres in 2015-16 to 12.94 lakh acres in 2021-22

The given information states that the horticulture crop sector contributes 31% to Agriculture GVA in
terms of value of the produce, with a value of Rs. 26,673 crores out of the total value of Rs. 85,959
crores of Agriculture & Allied activities during 2020-21.

74) Which of the following statements are true regarding Crop diversification?

1. Crop diversification is not recommended by researchers for maximizing farm income

2. Crop diversification helps farmers to gain higher farm income

3. The government does not recommend crop diversification

4. Crop diversification helps avoid glut in production


5. Oil Palm is not a recommended crop for diversification

Which of the following statements are true:

A) 2 only

B) 2,4, only

C) 1,2,3 only

D) 3only

Answer: B

Explanation : Researchers recommend crop diversification as a key strategy to maximize farm


income and make it more resilient to price fluctuations. The government has recognized the
importance of crop diversification and recommends certain crops like Oil Palm, Jowar, and Mustard.
Crop diversification also helps to avoid overproduction. The districts of Nirmal, Vikarabad, and
Rangareddy were found to have greater crop diversification compared to Peddapalli, Karimnagar,
and Suryapet.

75) Consider the following statements:

1. The Saagu-Baagu program and Area Estimation through Remote Sensing are initiatives that aim to
increase the productivity, efficiency, and sustainability of the agriculture sector.

2. The state has not been a significant contributor to the total FDI inflows in the agriculture service
sector of the country.

3. The GSVA by agriculture and allied activities in the state has increased by 186% since the
formation of the state.

Which of the following statements are true:

A) 2 only

B) 2,4, only

C) 1,3 only

D) 3only

Answer: C

Explanation :The GSVA by agriculture and allied activities had increased by 186% from 2014-15 to
2022-23

The state was the top contributor to the total FDI inflows in the agriculture service sector of the
country with a share of 26.32% between October 2019 to September 2021

government is deploying Artificial Intelligence and other technologies through pilots such as AI4AI-
Artificial Intelligence for Agricultural Innovation through the Saagu-Baagu program and Area
Estimation through Remote Sensing.
76) What is the purpose of e-NAM?

a) To promote uniformity in agriculture production

b) To integrate markets at the district level

c) To streamline marketing/transaction procedure and promote efficient functioning of the markets

d) To provide subsidies to farmers

Answer: c)

Explanation: e-NAM was launched with a vision to promote uniformity in agriculture marketing by
integrating markets at States across the country through a common online platform thus
streamlining marketing/transaction procedure and promoting efficient functioning of the markets.

77) Which commodities are traded on e-NAM?

a) Only food grains

b) Food grains, oilseeds, fruits and vegetables

c) Only fruits and vegetables

d) Only oilseeds

Answer: b)

Explanation: Around 175 commodities such as Food grains, Oilseeds, Fruits, and Vegetables are
traded on e-NAM.

78) Which Agricultural Market Committees (AMC) in Telangana were awarded the Prime Minister’s
Excellence award for successful implementation of e-NAM?

a) Nizamabad and Kesamudram

b) Karimnagar and Warangal

c) Adilabad and Nalgonda

d) Hyderabad and Rangareddy

Answer: a)

Explanation: The paragraph mentions that the AMC, Nizamabad and Kesamudram have been
awarded with the Prime Minister’s Excellence award for successful implementation of e-NAM.

79) Which startup incubated in T-Hub developed India's first private rocket?

a. Prarambh

b. Skyroot Aerospace

c. DPIIT
d. Vikram-S

Answer: b

Explanation: Skyroot Aerospace, a startup incubated in T-Hub, developed India's first private rocket
called "Prarambh".

80) Which sub-sector of Telangana's industrial sector had the highest CAGR between 2014-15 and
2021-22?

a. Manufacturing

b. Construction

c. Electricity and Utilities

d. None of the above

Answer: a

Explanation: The Manufacturing sub-sector of Telangana's industrial sector had a CAGR of 12.21%
between 2014-15 and 2021-22, which was the highest among the South Indian states.

81) Consider the following statements:

1. At the National Startup Awards 2022, T-Hub was able to outperform 55 other incubators at the
national level and was awarded the title of "Best Incubator in India".

2. In the Export Preparedness Index 2021, Telangana has scored 100 in the Business environment
indicator.

3. Telangana was included in the Top Achievers category in the latest edition (2020) of Ease of Doing
Business rankings.

Which of the following statements are true:

A) 2 only

B) 2,3, only

C) 1,2,3 only

D) 3only

Answer: C

Explanation: T-Hub won the "Best Incubator in India" award at the National Startup Awards 2022,
beating 55 other incubators. Telangana scored a perfect 100 in the Business environment indicator
of the Export Preparedness Index 2021 and was included in the Top Achievers category of the Ease
of Doing Business rankings in 2020.

82) What is Telangana's ranking in Ease of Doing Business (EoDB) since 2016?
A) Consistently ranked first

B) Consistently ranked second

C) Consistently ranked third

D) Consistently ranked among the top three

Answer: D)

Explanation: Consistently ranked among the top three 2016-1st ,2018-2nd,2019-3rd.

83) What is TS-iPASS Which was introduced in 2014 ?

A) A system for providing clearances to all projects within 60 days

B) A system for providing clearances to all projects within 30 days

C)A system for providing subsidies to industrial projects

D)A system for providing loans to industrial projects

Answer: B)

Explanation: TS-iPASS, introduced in 2014 in Telangana, provides clearance for all projects within 30
days and has reduced the administrative burden of setting up industries in the state, resulting in
consistently high rankings in Ease of Doing Business since 2016.

84) Which sub-sectors of Telangana's industrial sector have recorded higher Compound Annual
Growth Rate (CAGR) than the national level between 2014-15 and 2022-23?

a. Manufacturing and Mining

b. Electricity and Utilities and Mining

c. Manufacturing and Electricity and Utilities

d. Construction and Manufacturing

Answer: c.

Explanation :Telangana's industrial sub-sectors have experienced a greater Compound Annual


Growth Rate (CAGR) than the national average during the period of 2014-15 to 2022-23.

85) What is Lab32?

a) An incubation program for startups

b) A research laboratory for biotechnology

c) A coworking space for entrepreneurs

d) experimenting labs for seeds


Answer: a)

Explanation: Lab32 is the flagship incubation program of T-Hub, which has completed its eighth
cohort and has empowered over 240 startups from across the state. It provides various resources,
mentorship, and networking opportunities to startups.

86) Which startup from Telangana joined the unicorn club?

a) Skyroot Aerospace

b) Darwin Box

c) WhistleDrive

d) Samskar yatra

Answer: b)

Explanation: Darwin Box is a startup from Telangana that joined the unicorn club, a group of startups
with a valuation of over $1 billion. It provides human resources software and services to businesses.

387) What is the main objective of the Nethannaku Cheyutha scheme?

A. To provide social security to the weavers

B. To provide financial assistance to the weavers for purchasing raw materials

C. To provide subsidies for the weavers to be competitive in the market

D. To provide healthcare benefits to the weavers

Answer: A.

Explanation: The Nethannaku Cheyutha scheme is aimed at providing social security to the
handloom weavers of Telangana. The weavers contribute 8% of their wage into a savings account,
and the state contributes double the amount to provide social security to the weavers.

88) What is the Pavala Vaddi scheme?

A) A credit support scheme for the handloom weavers

B) An input subsidy scheme for the handloom weavers

C) A scheme to reduce the burden of interest on loans taken by the handloom weavers

D) A scheme to provide insurance benefits to the weavers

Answer: C)

Explanation: The Pavala Vaddi scheme is aimed at reducing the burden of interest on loans taken by
the handloom weavers. Under the scheme, the government disburses funds to cover interest
payments of weaver loans.
89) Where is the Kakatiya Mega Textile Park located?

A) Rajanna Sircilla District

B) Warangal

C) Hyderabad

D) Karimnagar

Answer: B)

Explanation: As mentioned in the given information, the Kakatiya Mega Textile Park is located in
Warangal, which is the highest cotton-growing district in the state.

90) What is the primary objective of the Apparel Park and Weaving Park?

A) To generate employment on a large scale

B) To promote tourism in the state

C) To provide alternate livelihood opportunities to farmers

D) To encourage IT companies to invest in the state

Answer: A)

Explanation: The primary objective of the Apparel Park and Weaving Park is to generate employment
on a large scale and provide alternate livelihood opportunities to women currently involved in beedi
rolling work.

91) Consider the following statements:

1. The Compounded Annual Growth Rate (CAGR) of Gross Value Added (GVA) by the services sector
in Telangana is higher than the All-India average

2. Real Estate, Ownership of Dwelling, and Professional Services sub-sector is the largest sub-sector
in terms of GVA contribution in Telangana

3. The contribution of the services sector to employment is higher in Telangana's rural areas than in
urban areas

Which of the following statements are not true:

A) 2 only

B) 2,4, only

C) 3 only

D) 2,3 only

Answer: C
Explanation: Telangana's services sector dominates its economy, contributing 62.81% of the Gross
State Value Added. The sub-sectors of Real Estate, Ownership of Dwelling, and Professional Services
and Trade, Repair, Hotels, and Restaurants have experienced significant growth rates. While the
services sector employs one-third of Telangana's workforce, it is more significant in urban areas than
in rural areas.

92) What is the Telangana Academy for Skill and Knowledge (TASK)?

a) A government program to enhance the skilling of engineering students in Telangana

b) An initiative to provide training in skill development, entrepreneurship, and capacity building

c) A program to provide higher education to students in Telangana

d) An organization to promote pre-primary education in Telangana

Answer: b)

Explanation: An initiative to provide training in skill development, entrepreneurship, and capacity


building.

93) With which partners has TASK collaborated for entrepreneurship and capacity building in 2021-
22?

a) Microsoft, Facebook, Amazon, and LinkedIn

b) Google Cloud, Oracle, Internshala, and Smartbridge

c) Airbnb, Uber, Ola, and Zomato

d) Samsung, LG, Sony, and Apple

Answer: b)

Explanation: Google Cloud, Oracle, Internshala, and Smartbridge are the companies which partners
has TASK collaborated for entrepreneurship and capacity building in 2021-22

94) Which Telangana heritage sites won awards in UNECO's Asia-Pacific Awards for Cultural Heritage
Conservation 2022?

a) Charminar and Warangal Fort

b) Golconda Fort step-wells and Domakonda Fort

c) Qutb Shahi Tombs and Falaknuma Palace

d) None of the above

Answer: b)

Explanation: In UNECO's Asia-Pacific Awards for Cultural Heritage Conservation 2022, Golconda Fort
step-wells won the award of distinction and Domakonda Fort in Kamareddy won the award of merit.
95) Consider the following statements:

1. The Medaram Jathara, also known as Sammakka Saralamma Jathara, is a tribal festival held every
two years to pay homage to the twin goddesses of Sammakka and Saralamma.

2. Since 2017, the government has been distributing Bathukamma sarees to all women who are
registered under the Food Security Scheme, during the festival season.

3. Bathukamma Sarees can be availed at every Mandal office in the state by all the women from
Telangana

Which of the following statements are not true:

A) 2 only

B) 2,4, only

C) 3 only

D) 3only

Answer: C

The Medaram Jathara is a biennial festival that pays tribute to the twin goddesses Sammakka and
Saralamma and is regarded as the largest tribal festival in Asia. Another annual tradition during the
festival season is the distribution of Bathukamma sarees by the government to all women who are
registered under the Food Security Scheme at only places the authority may decide.

96) Consider the following statements:

1. Telangana scored 100 in the Business environment indicator in the Export Preparedness Index
2021.

2. Telangana was ranked fifth among all landlocked states in the Export Preparedness Index 2021.

3. Telangana outperformed Uttar Pradesh and Rajasthan with a score of 1000, which was primarily
due to its strong innovation index performance and lower power cost.

Which of the following statements are true:

A) 2 only

B) 1,2,3 only

C) 1,2 only

D) None of the above

Answer: C

According to the Export Preparedness Index 2021, which was compiled by NITI Aayog and released
on March 25, 2022, Telangana was ranked as the fifth best landlocked state. In terms of the Business
environment indicator, Telangana outperformed Uttar Pradesh and Rajasthan with a score of 100,
which was primarily due to its strong innovation index performance and lower power cost.
97) What is the Telangana State Food Processing Society?

a) A government agency established in 2014 to promote the food processing industry in Telangana

b) An autonomous agency created in May 2014 to provide food processing equipment to farmers

c) An organization that helps farmers with food packaging and distribution

d) A regulatory agency that monitors food safety in Telangana

Answer: a)

Explanation: The Telangana State Food Processing Society is an autonomous agency created in May
2014 to promote and develop the food processing industry in the state.

98) Which of the following statements are true?

1. According to the Invest India portal maintained by the Government of India, Micro, Small and
Medium enterprises (MSMEs) account for 95% of all industrial units in India.

2. MSMEs are majorly concentrated in Hyderabad, Ranga Reddy, Medchal-Malkajgiri, Nalgonda,


Nizamabad, and Karimnagar districts, with Medak accounting for the highest share among all the
districts.

A) 2 only

B) 1 only

C) Both of them

D) None of the above

Answer: B

Explanation: According to Invest India, MSMEs make up 95% of all industrial units in India. MSMEs
are concentrated in several districts, with Hyderabad having the highest share among them.

99. Which sector employs more people than MSMEs in India?

a) Information technology

b) Healthcare

c) Agriculture

d) None of the above

Answer: c)

Explanation: MSMEs employ over 100 million people across the country, second only to the
agricultural sector.

100. What is the aim of Telangana Rythu Bhandu Samithi (TRBS) committees?
a) To provide financial assistance to farmers

b) To act as a bridge between farmers and Agriculture & Allied Departments

c) To construct multi-purpose cement ‘Kallams’

d) To facilitate farmers in sharing information on crops, markets, etc.

Answer: b)

Explanation: The Telangana government has formed Telangana Rythu Bhandu Samithi (TRBS)
committees at the village, Mandal, District, and State level. The total membership of these
committees is 1,60,990 persons who act as a bridge between farmers and Agriculture & Allied
Departments.

------------------------------------------

101. The value of goods and services produced by residents of a country and the value of their
property is called_________.

a. Gross domestic product

b. Net domestic product

c. Gross national income

d. Net national income

Answer: c

The value of goods and services produced by residents of a country and the value of their property is
called Gross national income. Gross domestic product of a country is the total of its net domestic
product and Consumption of capital in the production process

102. Which of the following ministries is responsible for the report on India’s national and per
capita income?

a. Ministry of Home Affairs

b. Ministry of Planning

c. Ministry of Statistics and Programme Implementation

d. Ministry of Human Resource Development

Answer: c

Ministry of Statistics and Programme Implementation is responsible for the report on India’s
national and per capita income. Ministry of Finance is responsible for computing the National
Income Statistics in India
103. Disguised unemployment exists primarily in the ________ sector in our country.

a. Defence

b. Agriculture

c. Manufacturing

d. None of the above

Answer: b

The meaning of disguised unemployment is Zero marginal productivity of labour.

When a large number of workers get unemployed due to automation of industrial processes, this
phenomenon is called Structural unemployment

104. The main characteristic of an underdeveloped economy is that it has a ________.

a. State of deprivation among large proportions of the population

b. High per capita income

c. A large proportion of the about force is in the tertiary sector

d. None of the above

Answer: a

The main characteristic of an underdeveloped economy is that it has a State of deprivation among
large proportions of the population Scarcity of capital, unemployment and technological
backwardness are generally found in Underdeveloped economies. The term economic growth is
explained by Structural changes in the economy, Increase in the per capita production and Increase
in the per capita income. Economic development can be explained as Improvement in the
distribution system, technology and production. A high proportion of labour in the primary sector is
an indicator of an economically developed nation

105. Starting from the financial year 2017-18, NITI Aayog has planned to replace the following

a. 5-year Vision Document

b. 10-year Vision Document

c. 15-year Vision Document

d. 20-year Vision Document

Answer: c

106. Consider the following statements regarding Growth and Development of the country:

1. Economic development is quantitative as well as qualitative progress in an economy.


2. The term growth means quantitative progress and the term development means
quantitative as well as qualitative progress.

Which of these statements are correct?

a. 1 Only

b. 2 Only

C. Both 1 and 2

d. None of the above

Answer: C

We can say that economic development is quantitative as well as qualitative progress in an


economy. It means, when we use the term growth we mean quantitative progress and when we use
the term development we mean quantitative as well as qualitative progress. If economic growth is
suitably used for development, it comes back to accelerate the growth and ultimately greater and
greater population brought under the arena of development.

Economic growth is quantitative progress. Growth is a value-neutral term. It might be positive or


negative for an economy for a specific period.

107. Consider the following statements regarding the Human Development Report (HDR):

1.This report was the first attempt to define and measure the level of development of economies.

2.United Nations Development Programme (UNDP) published its first Human Development Report
(HDR) in 1990.

Which of these statements is/are correct?

a. 1 Only

b. 2 Only

c. Both of them

d. neither of them

The report had a human development index (HDI) which was the first attempt to define and
measure the level of development of economies The dilemma of measuring the developmental level
of economies was solved once the United Nations Development Programme (UNDP) published its
first Human Development Report (HDR) in 1990.

The Index was a product of a select team of leading scholars, development practitioners and
members of the Human Development Report Office of the UNDP The first such team which
developed the HDI was led by Mahbub ul Haq and Inge Kaul

108. World happiness report considers which of the following parameters?

1. GDP per capita


2. Corruption

3. Education

Which of these statements is/are correct?

a. 1 and 2 Only

b. 2 and 3 Only

c 1and 3 only

d. All of the above

Answer: D

The nine domains of GNH are psychological well-being, health, time use, education, cultural diversity
and resilience, good governance, community vitality, ecological diversity and resilience, and living
standards.

New concept of assessing development, through the Gross National Happiness, was developed by
Bhutan. Without rejecting the idea of Human Development propounded by UNDP. Bhutan has been
officially following the targets set by the GNH.

109. Which of the following statements are correct about the Human Development Report 2021-
22

1. India’s rank on the Human Development Index (HDI) has slipped from 130 in 2020 to 132 in
2022

2. HDI is a composite index and measures Life expectancy at birth, Expected years of schooling,
Mean years of schooling, Gross national income (GNI) per capita

Which of these statements is/are correct?

a. 1 Only

b. 2 Only

c. Both of them

d. neither of them

Answer: C

HDI is a composite index that measures average achievement in human development taking into
account four indicators:

o Life expectancy at birth (Sustainable Development Goal 3),

o Expected years of schooling (Sustainable Development Goal 4.3),

o Mean years of schooling (Sustainable Development Goal 4.4),

o Gross national income (GNI) (Sustainable Development Goal 8.5).


110. The Multi-dimensional Poverty Index developed by Oxford Poverty and Human Development
Initiative with UNDP support covers which of the following

1. Deprivation of education, health, assets and services at household level

2. Purchasing power parity at national level

3. Extent of budget deficit and GDP growth rate at national level

Select the correct answer using the codes given below:

(a) 1 only

(b) 2 and 3 only

(c) 1 and 3 only

(d) 1, 2 and 3

Ans: (a)

111. Match the following:

1. Earth Summit- 2012

2. World Summit on Sustainable Development 1992

3. United Nations Conference on Sustainable Development 2002

Select the correct choice:

a. 1 and 2 only

b. 2 and 3 only

c. 1, 2 and 3

d. None of the above

Ans: D

Earth Summit 1992 is also known as The United Nations Conference on Environment and
Development (UNCED).190 countries pledged their commitment to achieve by 2010, a significant
reduction in the current rate of biodiversity loss at global, regional and local levels.As a follow-up,
the World Summit on Sustainable Development (Rio+10) was held in 2002 in Johannesburg, South
Africa.In 2012, the United Nations Conference on Sustainable Development was also held in Rio and
is also commonly called Rio+20 or Rio Earth Summit 2012.

112. Match the following:

1. Rio Declaration: principles intended to guide countries in future sustainable development.


2. Agenda 21: non-binding action plan of the United Nations with regard to sustainable
development.

3. Forest Principles: Non-legally binding document on Conservation and Sustainable


Development of All Types of Forests

Which of the above are matched correctly?

a. 1 and 2 only

b. 1 and 3 only

c. 2 and 3 only

d. All of the above

Ans: D

Rio Declaration: principles intended to guide countries in future sustainable development.

Agenda 21: non-binding action plan of the United Nations with regard to sustainable development.

Forest Principles: Non-legally binding document on Conservation and Sustainable Development of All
Types of Forests.

113. Select the correct statements regarding the sustainable development goals (SDGs):

1. The UN 2030 Agenda for Sustainable Development was launched in 2005

2. The UN 2030 Agenda’s Sustainable Development Goals (SDG) aim at eradicating poverty in
all forms and “seek to realize the human rights of all and achieve gender equality”.

3. The Sustainable Development Goals (SDGs) are a collection of 17 global goals, and their 169
targets, set by the United Nations General Assembly in 2015 for the year 2030 (UNGA resolution
“2030 Agenda”).

Select the correct choice:

a. 1 and 2 only

b. 1 and 3 only

c. 2 and 3 only

d. All of the above

Ans: c

1. The UN 2030 Agenda for Sustainable Development was launched in 2015

2. The UN 2030 Agenda’s Sustainable Development Goals (SDG) aim at eradicating poverty in
all forms and “seek to realize the human rights of all and achieve gender equality”.

3. The Sustainable Development Goals (SDGs) are a collection of 17 global goals, and their 169
targets, set by the United Nations General Assembly in 2015 for the year 2030 (UNGA resolution
“2030 Agenda”).
114. With reference to the Agreement at the UNFCCC Meeting in Paris in 2015, which of the
following statements is/are correct?

1) The Agreement was signed by all the member countries of the UN and it will go into effect in
2017.

2) The Agreement aims to limit the greenhouse gas emissions so that the rise in average global
temperature by the end of this century does not exceed 2 °C or even 1.5 °C above pre-industrial
levels.

3) Developed countries acknowledged their historical responsibility in global warming and


committed to donate $ 1000 billion a year from 2020 to help developing countries to cope with
climate change.

Select the correct answer using the code given below.

a) 1 and 3 only

b) 2 only

c) 2 and 3 only

d) 1, 2 and 3

Answer: b) 2 only

• The Agreement was signed by all the member countries of the UN and entered into effect in
November 2016

• Developed countries acknowledged their historical responsibility in global warming and


committed to donate $ 100 billion a year from 2020 to help developing countries to cope with
climate change.

115.The term ‘Intended Nationally Determined Contributions’ is sometimes seen in the news in the
context

of

a) pledges made by the European countries to rehabilitate refugees from the war-affected Middle
East

b) plan of action outlined by the countries of the world to combat climate change

c) capital contributed by the member countries in the establishment of Asian Infrastructure


Investment Bank

d) plan of action outlined by the countries of the world regarding Sustainable Development Goals

Answer: b)

India has presented the following five nectar elements (Panchamrit) of India’s climate action:

1. Reach 500 GW Non-fossil energy capacity by 2030.


2. 50 per cent of its energy requirements from renewable energy by 2030.

3. Reduction of total projected carbon emissions by one billion tonnes from now to 2030.

4. Reduction of the carbon intensity of the economy by 45 per cent by 2030, over 2005 levels.

5. Achieving the target of net zero emissions by 2070.

116.Consider the following Acts:

1. The Water (Prevention and Control of Pollution) Act, 1974

2. Water (Prevention and Control of Pollution) Cess Act, 1977

3. Forest (Conservation) Act, 1980

4. Biological Diversity Act, 2002

5. Public Liability Insurance Act, 1991

6. Environment (Protection) Act, 1986

7. Air (Prevention and Control of Pollution) Act, 1981

Which of the above are under the jurisdiction of the National Green Tribunal?

a. 2, 3, and 4 only

b. 3, 4 and 5 only

c. 3,4,5,6 and 7 only

d. 1,2,3,4,5,6,7

Answer: D

Key Points National Green Tribunal (NGT):

• It was constituted on October 18, 2010, by the National Green Tribunal (NGT Act of 2010).

• It is a specialist body tasked with resolving environmental disputes including


multidisciplinary difficulties.

• It took the place of the National Environment Appellate Authority. • The Tribunal's principal
place of sitting is in New Delhi, while the other four places of sitting are Bhopal, Pune, Kolkata, and
Chennai.

• The NGT is required to issue a final decision on petitions or appeals within six months of
their submission.

Jurisdiction of NGT:

The NGT has the power to hear all civil cases relating to environmental issues and questions that are
linked to the implementation of laws listed in

Schedule I of the NGT Act.


These include the following:

The Water (Prevention and Control of Pollution) Act, 1974; o The Water (Prevention and Control of
Pollution) Cess Act, 1977;

o The Forest (Conservation) Act, 1980;

o The Air (Prevention and Control of Pollution) Act, 1981;

o The Environment (Protection) Act, 1986;

o The Public Liability Insurance Act, 1991;

o The Biological Diversity Act, 2002. Hence all are correct.

The NGT has not been given the authority to consider cases pertaining to the Wildlife (Protection)
Act, 1972, the Indian Forest Act, 1927, and several state legislation dealing with forests, and tree
preservation.

117.How is the National Green Tribunal (NGT) different from the Central Pollution Control Board
(CPCB)

1) The NGT has been established by an Act whereas the CPCB has been created by an executive
order of

the Government.

2) The NGT provides environmental justice and helps reduce the burden of litigation in the higher
courts

whereas the CPCB promotes cleanliness of streams and wells and aims to improve the quality of air
in the

country.

Which of the statements given above is/are correct?

a) 1 only

b) 2 only

c) Both 1 and 2

d) Neither 1 nor 2

Answer: b) 2 only

• CPCB is a statutory organisation under the MoEFC.

• It was established under the Water (Prevention and Control of Pollution) Act, 1974.

118. In the cities of our country, which among the following atmospheric gases are normally
considered in calculating the value of Air Quality Index?

1) Carbon dioxide
2) Carbon monoxide

3) Nitrogen dioxide

4) Sulphur dioxide

5) Methane

Select the correct answer using the code given below.

a) 1, 2 and 3 only

b) 2, 3 and 4 only

c) 1, 4 and 5 only

d) 1, 2, 3, 4 and 5

Answer: b) 2, 3 and 4 only

The AQI will consider eight pollutants (PM10, PM2.5, NO2, SO2, CO, O3, NH3, and Pb).

119.In the context of solving pollution problems, what is/are the advantage/advantages of
bioremediation technique?

1) It is a technique for cleaning up pollution by enhancing the same biodegradation process that
occurs in

nature.

2) Any contaminant with heavy metals such as cadmium and lead can be readily and completely
treated by

bioremediation using microorganisms.

3) Genetic engineering can be used to create microorganisms specifically designed for


bioremediation.

Select the correct answer using the code given below:

a) 1 only

b) 2 and 3 only

c) 1 and 3 only

d) 1, 2 and 3

Answer: c) 1 and 3 only

• Bioremediation is limited to biodegradable compounds. Bioremediation is the use of


microorganisms (bacteria and fungi) to degrade the environmental contaminants into less toxic
forms.

• Microorganisms can be specifically designed for bioremediation using genetic engineering


techniques.
120. Due to improper/indiscriminate disposal of old and used computers or their parts, which of
the following are released into the environment as e-waste?

1. Beryllium

2. Cadmium

3. Chromium

4. Heptachlor

5. Mercury

6. Lead

7. Plutonium

Select the correct answer using the codes given below.

a) 1, 3, 4, 6 and 7 only

b) 1, 2, 3, 5 and 6 only

c) 2, 4, 5 and 7 only

d) 1, 2, 3, 4, 5, 6 and 7

Explanation:

Answer: b) 1, 2, 3, 5 and 6 only

• Heptachlor is a Chlorohydrocarbon (CHC) which is used as an insecticide.

• Plutonium is a radioactive metal and hence not used in computers.

121. Which of the following acts doesn’t fall in the category of Social Security Legislation?

1. The EPFO ACT 1952

2. The Workmen’s Compensation Act 1923

3. The Trade Union Act 1926

4. The Industrial Disputes Act 1947

Select the correct code

A. Only 1

B. Only 2

C. Both 1 &2

D. Both 3&4

Answer: D

Explanation:
Social security legislation in India in the Industrial sector consists of the enactments: (1) The
workmen's Compensation Act, 1923; (2) The Employees State Insurance Act, 1948; (3) The
Employees' Provident Funds and Miscellaneous Provisions Act, 1952; (4) The Maternity Benefit Act,
1961; and (5) The Payment of Gratuity Act, 1970.

122. HDI is an aggregate measure of progress in which of the three dimensions?

A. Health, Education, Standard of living

B. Food Security, Employment,Income

C. Agriculture,Industry, Services

D. None

Answer: A

Explanation:

123. Which of the following is the amount of annual premium of the Pradhan Mantri Suraksha
Bima Yojana(PMSBY) for accident and disability cover up to 2,00,000 ?

A. 100

B. 50

C. 20

D. 12

Answer: C

Explanation:

Pradhan Mantri Suraksha Bima Yojana is a government scheme launched on 9th May 2015 by PM
Narendra Modi.

A government-sponsored accident insurance program, the Pradhan Mantri Suraksha Bima Yojana,
provides coverage for accidental death, total and partial disability, as well as permanent disability. A
person must be between the ages of 18 and 70 to apply for this program. Anyone between the ages
of 18 and 70 may participate in this program, but they must have a bank account. The scheme's
yearly premium, without including service tax, is Rs.20. The premium payment is automatically
deducted from the scheme holder's bank account. If the subscriber is totally incapacitated or dies in
an accident, Rs.2 lakh is paid to the nominee. If the subscriber has an accident and experiences a
partial permanent disability, they will receive Rs.1 lakh.

124. With reference to National Investment and Infrastructure Fund Limited (NIIFL) which of
the following statements are correct

1. The Governing Council of the National Investment and Infrastructure Fund (NIIF) is chaired
by the Finance Minister of India
2. NIIF manages only Master Fund, Strategic Fund.

Choose the correct code

A. 1 only

B. 2 only

C. Both

D. None

Answer: A

Explanation: The Governing Council of the National Investment and Infrastructure Fund (NIIF) is
chaired by the Hon’ble Finance Minister of India and has members from corporate bodies,
investments and policy sectors. The Governing Council meets once a year and provides suggestions
of the overall strategic decisions of the NIIFL. The Governing Council is currently chaired by Finance
Minister of India Nirmala Sitharaman , Finance Minister of India.] Among the other council members
are Debasish Panda (Secretary of Financial Services), Ajay Seth (Secretary, Department of Economic
Affairs), Dinesh Khara (Chairman, State Bank of India), Hemendra Kothari (Chairman, DSP Group), T.
V. Mohandas Pai (Chairman, Manipal Global Education).

NIIF manages three funds: Master Fund, Fund of Funds and Strategic Fund. The funds were set up to
make investments in India by raising capital from domestic and international institutional investors.

125. Regarding 'Atal pension Yojana', which of the following statements is/are correct?

1. It is a minimum guaranteed pension scheme mainly targeted at unorganized sector workers.

2. From October 1, 2022, the government has banned income taxpayers from investing in the Atal
Pension Yojana

3. Same amount of pension is guaranteed for the spouse for life after subscriber's death.

Select the correct answer using the code given below.

A. Only 1

B. Only 2 and 3

C. Only 1 and 3

D. All the above

Answer: D

Explanation:

The Yojana mainly addresses the longevity risks among the workers in unorganised sector and
encourages them to voluntarily save for their retirement. So, 1 is correct. Atal Pension Yojana (APY)
is open to all bank account holders who are not members of any statutory social security scheme. As
per the Ministry of Finance, “The spouse of the subscriber shall be entitled to receive the same
pension amount as that of the subscriber until the death of the spouse. After the death of both the
subscriber and the spouse, the nominee of the subscriber shall be entitled to receive the pension
wealth, as accumulated till age of 60 years of the subscribe”. So, clearly 3 is correct. From October 1,
2022, the government has banned income taxpayers from investing in the Atal Pension Yojana (APY).
So all the options are correct.

126. Consider the following statements

A. As per the Census, sex ratio has increased from 933 in 2001 to 943 females per thousand
males in 2011

B. As per the 2011 Census, Kerala has the highest sex ratio compared to other states.

Choose the correct code

A. 1 only

B. 2 only

C. Both

D. None

Answer: C

Explanation:

The Sex Ratio in the country has shown an improvement. As per the Census, sex ratio has increased
from 933 females per thousand males in 2001 to 943 females per thousand males in 2011. State/UT-
wise details of sex ratio are annexed. As per the 2011 Census, Kerala has the highest sex ratio of
1037 females per thousand males.

127. Choose the correct answer

1. The share of self-employed increased and that of regular wage/salaried workers declined.

2. The share of casual labour increased slightly, driven by rural areas.

A. 1 only

B. 2 only

C. Both

D. None

Answer: A
Explanation:

According to broad status in employment, the share of self-employed increased and that of
regular wage/salaried workers declined in 2020-21 vis-à-vis 2019-20, driven by trend in both rural
and urban areas. The share of casual labour declined slightly, driven by rural areas.

128.

1. A three-day Akhil Bharatiya Shiksha Samagam was organised at Varanasi on 7-9 July 2022 by the
Ministry of Skill Development and Entrepreneurship

2. The Samagam was witness to India’s extended vision and a renewed commitment to help achieve
the goals of the higher education system.

Choose the incorrect option

A. 1 only

B. 2 only

C. Both

D. None

Answer: A

Explanation:

A three-day Akhil Bharatiya Shiksha Samagam was organised at Varanasi on 7-9 July 2022 by the
Ministry of Education in association with the UGC and Banaras Hindu University. The event brought
together over 300 Vice Chancellors and Directors from public and private universities, educationists,
policymakers, as also industry representatives to deliberate on how the implementation of NEP 2020
can be taken further across the country after successful implementation of several initiatives in the
last two years. The Summit provided a platform for thought-provoking discussions that helped
articulate the roadmap and implementation strategies, foster knowledge exchange, build networks
through interdisciplinary deliberations, and discuss challenges being faced by educational
institutions and articulate solutions. The Samagam was witness to India’s extended vision and a
renewed commitment to help achieve the goals of the higher education system with education
leaders resolving to work collectively for transforming India into an equitable and vibrant knowledge
society

129. Consider the following statements regarding the socioeconomic welfare measures proposed by
the government:

1. The payment received from the Agniveer Corpus Fund by the Agniveers enrolled in Agnipath
Scheme, 2022 is proposed to be exempt from taxes.
2. It is proposed to enable co-operatives to withdraw cash up to Rs 5 crore in a year without being
subjected to TDS on such withdrawal.

3. Certain start-ups are eligible for some tax benefit if they are incorporated before 1st April, 2023,
the period of incorporation is proposed to be extended by one year to before 1st April, 2024.

Choose the correct option using the codes below:

A. 1 and 2 only

B. 1 and 3 only

C. 2 and 3 only

D. 1, 2 and 3

Answer: B

Explanation:

The payment received from the Agniveer Corpus Fund by the Agniveers enrolled in Agnipath
Scheme, 2022 is proposed to be exempt from taxes. Deduction in the computation of total income is
proposed to be allowed to the Agniveer on the contribution made by him or the Central Government
to his Seva Nidhi account.

It is proposed to enable co-operatives to withdraw cash up to ` 3 crore in a year without being


subjected to TDS on such withdrawl.

Certain start-ups are eligible for some tax benefit if they are incorporated before 1st April, 2023. The
period of incorporation of such eligible start-ups is proposed to be extended by one year to before
1st April, 2024.

130. According to the Union Budget 2023-24, consider the following statements.

1. Government will recruit 38,800 teachers and support staff for the 740 Eklavya Model Residential
Schools, serving 3.5 lakh tribal students.

2. The outlay for PM Awas Yojana is being enhanced by 66 per cent to over ` 79,000 crore. Which of
the above statements is/are correct?

A. 1 only

B. 2 only

C. Both 1 and 2

D. None of the above

Answer: C
Explanation:

In the next three years, centre will recruit 38,800 teachers and support staff for the 740 Eklavya
Model Residential Schools, serving 3.5 lakh tribal students

The outlay for PM Awas Yojana is being enhanced by 66 per cent to over ` 79,000 crore.

131. Consider following wrt measurement of unemployment in India

A. NSSO estimates unemployment rates in India.

B. The daily status unemployment is considered to be a comprehensive measure of


unemployment

C. Usual status unemployment gives highest estimates of unemployment.

D. All

Answer: D

Explanation:

• There are three measures or estimates of unemployment. These are developed by National
Sample Survey Organisation (NSSO). They are:

Usual Status Unemployment

This measure estimates the number of persons who remained unemployed for a major part of the
year.This measure gives the lowest estimates of unemployment

Weekly Status Unemployment

The estimate measures unemployment with respect to one week. A person is said to be unemployed
if he is not able to work even for an hour during the survey period. In other words according to this
estimate, a person is said to be employed for the week, even if he/she is employed only for a day
during that week

Current Daily Status Unemployment

It considers the activity status of a person for each day of the preceding seven days. The reference
period here is a day. If a person did not find work on a day or some days during the survey week,
he/she is regarded as unemployed. Normally if a person works for four hours or more during a day,
he or she is considered as employed for the whole day. The daily status unemployment is considered
to be a comprehensive measure of unemployment.

132. Choose correct one

A. National Food for Work Programme was launched on November 14, 2004

B. The programme is open to all urban poor who are in need of wage employment and desire
to do manual unskilled work.
C. Both

D. None

Answer: A

Explanation:

National Food for Work Programme was launched on November 14, 2004 in 150 most backward
districts of the country with the objective to intensify the generation of supplementary wage
employment. The programme is open to all rural poor who are in need of wage employment and
desire to do manual unskilled work. It is implemented as a 100 per cent centrally sponsored scheme
and the food grains are provided to States free of cost. However, the transportation cost, handling
charges and taxes on foodgrains are the responsibility of the States. The collector is the nodal officer
at the district level and has the overall responsibility of planning, implementation, coordination,
monitoring and supervision.

133. Consider statements wrt MGNREGA

A. it guarantees at least 100 days of wage employment in each financial year to every
household that is willing to do unskilled manual labour.

B. The act has its roots in the Maharashtra Employment Guarantee Scheme (MEGS), 1972

C. It is the gram panchayat’s responsibility to allocate work within 15 days of the request and
provide wages within 10 days of job completion.

Choose correct one

1.A and B 2. A only

3.A and C 4. All

Answer: 1

Explanation:

The National Rural Employment Guarantee Act (NREGA) was passed by the Parliament of India in
2005 and came into effect on February 2, 2006, as a social and legal measure that guarantees the
‘right to work’. Renamed Mahatma Gandhi National Rural Employment Guarantee Act (MGNREGA)
in 2009, it guarantees at least 100 days of wage employment in each financial year to every
household that is willing to do unskilled manual labour. The act has its roots in the Maharashtra
Employment Guarantee Scheme (MEGS), 1972, which was the first to recognise the right to work,
and the success of which paved the way for MGNREGA.

The gram panchayat is the main facilitator for households that wish to apply for work under
MGNREGA. The first step that an unregistered household must take is to apply for a job card through
their gram panchayat. Once they are issued a job card post verification of their documents and
confirmation of their eligibility, they have to submit an application for employment. It is the gram
panchayat’s responsibility to allocate work within 15 days of the request and provide wages within
15 days of job completion.

134. If a person is ready to work at the prevailing wage rate in the market, but he is unable to find
the work, then what type of unemployment would it be called?

A. Voluntary unemployment

B. Involuntary unemployment

C. Seasonal unemployment

D. None of the above

Answer: B

Explanation:

Involuntary unemployment is a type of unemployment in which the person wishes to work but
he/she does not get the job because the country's economy lacks in providing one.

135. ______unemployment rate is the minimum unemployment rate resulting from real or
voluntary economic forces.

A. Frictional

B. Structural

C. Natural

D. Voluntary

Answer: C

Explanation:

The natural unemployment rate is the minimum unemployment rate resulting from real or voluntary
economic forces. It represents the number of people unemployed due to the structure of the labour
force, such as those replaced by technology or those who lack the skills to get hired.

Natural unemployment is commonplace in the labour market as workers flow to and from jobs or
companies. Unemployment is not considered natural if it is cyclical, institutional, or policy-based
unemployment.

Because of natural unemployment, 100% full employment is unattainable in an economy.

136. Consider following statements wrt DAY-NRLM

A. Scheme partially funded by Asian development bank.


B. The scheme is an improved version of the earlier Swarnjayanti Gram Swarozgar Yojana
(SGSY).

C. Mahila Kisan Sashaktikaran Pariyojana (MKSP) sub-scheme is to empower women in


agriculture by making systematic investments to enhance their participation and productivity.

Options: 1. A only 2. A and B

3.A and C 4. B and C

Answer: 4

Explanation:

The DAY-NRLM is essentially a poverty relief programme of the Central government. It was launched
as ‘Aajeevika – National Rural Livelihoods Mission (NRLM)’ by the GOI’s Ministry of Rural
Development in the year 2011. It was renamed as DAY-NRLM in 2015.

The scheme is an improved version of the earlier Swarnjayanti Gram Swarozgar Yojana (SGSY).

The programme is supported partially by the World Bank.

NRLM Mission is to “To reduce poverty by enabling the poor households to access gainful self-
employment and skilled wage employment opportunities, resulting in an appreciable improvement
in their livelihoods on a sustainable basis, through building strong grassroots institutions of the
poor.”

Mahila Kisan Sashaktikaran Pariyojana (MKSP)

The main objective of this sub-scheme is to empower women in agriculture by making systematic
investments to enhance their participation and productivity. The programme also seeks to create
and sustain agriculture-based livelihoods concerning women in rural areas. Other aims are to ensure
food and nutrition in the households, enable better access to services and inputs for women,
improve the managerial capabilities of women, etc.

137. Choose correct one

A. LFPR is defined as the percentage of persons in labour force (i.e. working or seeking or
available for work) in the population.

B. According to PLFS report 2021-22 there is slight decrease in female LFPR.

C. 55.2% is the LFPR of India during 2021-22

Options: 1. A only 2. A and C

3.A and B 4. B only

Answer: 2

explanation:
138. Which of the following statements is true about unemployment?

a. Seasonal unemployment occurs when there are more people working in a job than is
actually required

b. Open unemployment occurs when there are more people working in a job than is actually
required

c. Disguised unemployment occurs when there are more people working in a job than is
actually required

d. Educated unemployment occurs when there are more people working in a job than is
actually required

Answer: C

Explanation:

Seasonal unemployment occurs when people are unemployed at particular times of the year when
demand for labour is lower than usual. Examples are agricultural labours and construction workers.

Open unemployment is the situation when a person is willing to work, is educated but is unable to
get a job and work.

Educated unemployment- when an educated people do not find jobs it is called educated
unemployment. The most concerning reason for unemployment or educated unemployment is the
population growth in the Indian economy.

139. Consider following and choose correct one

a. The State of Inequality in India Report was released today by Dr Bibek Debroy, Chairman,
Economic Advisory Council to the Prime Minister

b. The share of the top 1% accounts for 6-7% of the total incomes earned, while the top 10%
accounts for one-third of all incomes earned.

c. Both

d. None

Answer: C

Explanation:

The State of Inequality in India Report was released today by Dr Bibek Debroy, Chairman, Economic
Advisory Council to the Prime Minister (EAC-PM). The report has been written by the Institute for
Competitiveness and presents a holistic analysis of the depth and nature of inequality in India. The
report compiles information on inequities across sectors of health, education, household
characteristics and the labour market. As the report presents, inequities in these sectors make the
population more vulnerable and triggers a descent into multidimensional poverty.

The report moves beyond the wealth estimates that depict only a partial picture to highlight
estimates of income distribution over the periods of 2017-18, 2018-19 and 2019-20. With a first-
time focus on income distribution to understand the capital flow, the report emphasises that wealth
concentration as a measure of inequality does not reveal the changes in the purchasing capacity of
households. Extrapolation of the income data from PLFS 2019-20 has shown that a monthly salary of
Rs 25,000 is already amongst the top 10% of total incomes earned, pointing towards some levels of
income disparity. The share of the top 1% accounts for 6-7% of the total incomes earned, while the
top 10% accounts for one-third of all incomes earned. In 2019-20, among different employment
categories, the highest percentage was of self-employed workers (45.78%), followed by regular
salaried workers (33.5%) and casual workers (20.71%). The share of self-employed workers also
happens to be the highest in the lowest income categories.

140. “Poverty and Shared Prosperity 2022: Correcting Course” report released by

a. World bank

b. IMF

c. OXFAM

d. UNDP

ANSWER: A

Explanation:

Recently, the World Bank released a report titled “Poverty and Shared Prosperity 2022: Correcting
Course”

141. As per the latest Annual Survey of Industries 2019-20, the percent of persons in the
registered manufacturing sector were employed in the food processing sector is?

A) 10.1 percent

B) 15.9 percent

C) 12.2 percent

D) 9.0 percent

Ans: C

The food processing sector is of enormous significance for India's development because of the
strong connections and interactions it promotes between industry and agriculture. During the last
five years ending FY21, the food processing industries sector has been growing at an average annual
growth rate of around 8.3 per cent. As per the latest Annual Survey of Industries (ASI) 2019-20, 12.2
per cent of persons in the registered manufacturing sector were employed in the food processing
sector. The value of agri-food exports, including processed food exports, was about 10.9 per cent of
India's total exports during 2021-22.

142. Liquid Nano Urea has been developed and patented by which among the following?

A) Department of Agriculture and Farmers Development


B) IFFCO

C) NITI Aayog

D) Department of Economic Affairs

Ans: B

Liquid Nano Urea has been developed and Patented by IFFCO. IFFCO Nano Urea is the only Nano
fertilizer approved by the Government of India and included in the Fertilizer Control Order (FCO).
Liquid Nano Urea contains 4% nitrogen by volume and comes in the form of nanoparticle. Its
efficiency is 85%-90% as compared to 25% for conventional urea Unutilized nitrogen is stored in the
plant vacuole and is slowly released for proper growth and development of the plant. Small size (20-
50 nm) of Nano Urea increases its availability to crop by more than 80%.

143. Assertion (A): Pradhan Mantri Kisan Samman Nidhi (PM-KISAN) scheme a Centrally
sponsored scheme.

Reason (R): Under the Pradhan Mantri Kisan Samman Nidhi scheme an income support of Rs.8000/-
per year is provided to all eligible farmer families across the country in four equal instalments of
Rs.2000/- every three months.

Select the answer using the codes given below

A) Both A and R are true, and R is proper explanation of A

B) Both A and R are true, and R is not proper explanation of A

C) Only A is true

D) Only R is true

Ans: C

Pradhan Mantri Kisan Samman Nidhi (PM-KISAN) scheme is effective from 1.12.2018. It is a Central
Sector scheme with 100% funding from Government of India. Under the Scheme an income support
of Rs.6000/- per year is provided to all eligible farmer families across the country in three equal
instalments of Rs.2000/- every four months. Definition of family for the Scheme is husband, wife and
minor children. The entire responsibility of identification of beneficiary farmer families rests with the
State / UT Governments. Some farmers have been excluded from this scheme for example,
employees of central and state government, PSU employees, pensioners etc.

144. ‘PUSA Decomposer’ was seen sometimes in the news, associated with?

A) Liquid Nano urea

B) Farmer’s friendly cold storage complex in Rajasthan

C) Paddy straw decomposer

D) Straw less wheat developed by ICAR

Ans: C
The Govt. is also testing "PUSA Decomposer", which will help the paddy straw to decompose at a
much faster rate than usual, giving the farmers the option to shred the straw, spray a SOLUTION
containing the fungal strains, and mix it with the soil for decomposition. If it becomes successful, it
will be a new revolution in farming which will both reduce air pollution and increase soil fertility.]

145. The Production Linked Incentive Scheme for Food Processing Industry (PLISFPI) was
launched in?

A) January 2022

B) January 2023

C) March 2023

D) March 2022

Ans: D

The Production Linked Incentive Scheme for Food Processing Industry (PLISFPI), launched in March
2022, has the specific mandate to incentivise investments to create global food champions. Sectors
with high growth potential, like marine products, processed fruits & vegetables, and ‘Ready to Eat/
Ready to Cook’ products, are covered for support. In phase -I, 149 applications have been selected
for assistance under the PLISFPI.

146. Consider the following statements regarding the farmers producers’ organizations

i) The farmers producers’ organizations were only a cooperative society.

ii) The main aim of farmers producers’ organizations is to ensure better income for the farmers
through an organization of their own.

Which of the statements given above are correct?

A) Only i

B) Only ii

C) Both

D) None

Ans: B

Indian agriculture is dominated by marginal and small farmers, who suffer serious disadvantage in
terms of scale, uneconomic lot for marketing and price risk. Small sized farmers are also
disadvantaged in terms of bargaining power in various transactions in the input and output markets.
These handicaps can be overcome by organizing farmers under some institutional mechanism like
the farmers producers’ organizations (FPOs). FPOs can be a company, a cooperative society, Trust or
any other form of legal entity which provides for sharing of profits/benefits among the farmers.
Ownership control is always with the members/farmers and management is through the
representatives of the members. The main aim of an FPO is to ensure better income for the farmers
through an organization of their own.
147. The Main reasons for low productivity in Indian agriculture are?

i) Population pressure,

ii) uneconomic (small) land holdings,

iii) uncertain monsoon,

iv) cooperative farming,

Select the answer using the codes given below

A) Only I, ii and iii

B) Only iii and iv

C) Only ii, iii and iv

D) All the above

Ans: A

Main reasons for low productivity in Indian agriculture are: Population pressure, uneconomic (small)
land holdings, uncertain monsoon and inadequate irrigation facilities, subsistence nature of farming,
disguised unemployment, decline in soil fertility, lack of resources etc., Co-operative farming is not a
reason for low productivity in Indian agriculture. In fact, co-operative farming leads to better
productivity.

148. According to the 2011 Census, the total work force of the country is associated with the
agriculture sector is?

A) 50 percent

B) 52 percent

C) 55 percent

D) 58 percent

Ans: C

As per 2001 Census, 58.2% of Indian labour force was connected to agriculture sector. According to
the 2011 Census, about 55% of the total work force of the country is associated with the agriculture
sector.

149. Consider the following statements regarding the Green Revolution

i) Green Revolution was introduced in India in early 1960.

ii) Green Revolution was introduced first for rice.

Which of the statements given above are correct?


A) Only i

B) Only ii

C) Both

D) None

Ans: A

Green Revolution is the introduction of new techniques of agriculture, which became popular by the
name of Green Revolution (GR) in early 1960s—at first for wheat and by the next decade for rice,
too. It revolutionised the very traditional idea of food production by giving a boost by more than 250
per cent to the productivity level. The Green Revolution was centred around the use of the High
Yielding Variety (HYV) of seeds developed by the US agro-scientist Norman Borlaug doing research
on a British Rockfellor Foundation Scholarship in Mexico by the early 1960s.

150. Consider the following statements regarding green revolution

i) GM Seeds only

ii) The Chemical Fertilizers

iii) The Irrigation

Which of the above are the components of the green revolution?

A) Only I and ii

B) Only ii and iii

C) Only iii

D) All the above

Ans: B

The Green Revolution was based on the timely and adequate supply of many inputs/components. A
brief review on the Green Revolution is given below: The HYV Seeds These seeds were popularly
called the ‘dwarf’ variety of seeds. With the help of repeated mutations, Mr. Borlaug had been able
to develop a seed which was raised in its nature of nutrients supplied to the different parts of the
wheat plant—against the leaves, stem and in favour of the grain. This made the plant dwarf and the
grain heavier—resulting in high yield. These seeds were non-photosynthetic, hence non-dependent
on sun rays for targeted yields. The Chemical Fertilizers The seeds were to increase productivity
provided they got sufficient level of nutrients from the land. The level of nutrients they required
could not be supplied with the traditional composts because they have low concentration of
nutrients content and required bigger area while sowing—it meant it will be shared by more than
one seed. That is why a high concentration fertiliser, were required, which could be given to the
targeted seed only—the only option was the chemical fertilisers—urea (N), phosphate (P) and
potash (K). The Irrigation For controlled growth of crops and adequate dilution of fertilizers, a
controlled means of water supply was required.

You might also like